Eigen values of a Third Order Linear Homogenous ODE












0












$begingroup$


I have two third order linear ODE which have been arrived after applying separation of variables to a system of PDEs



begin{eqnarray}
lambda_h F''' - 2 lambda_h beta_h F'' + left( (lambda_h beta_h - 1) beta_h - mu right) F' + beta_h^2 F &=& 0,\
V lambda_c G''' - 2 V lambda_c beta_c G'' + left( (lambda_c beta_c - 1) V beta_c + mu right) G' + V beta_c^2 G &=& 0,
end{eqnarray}



$F$ is $F(x)$ and $G$ is $G(y)$. The boundary conditions are



For $F$:
$$F(0)=0$$
$$frac{F''(0)}{F'(0)}=beta_h$$
$$frac{F''(1)}{F'(1)}=beta_h$$



For $G$:
$$G(0)=0$$
$$frac{G''(0)}{G'(0)}=beta_c$$
$$frac{G''(1)}{G'(1)}=beta_c$$



$lambda_h$, $lambda_c$, $beta_h$ and $beta_c$ are all constants $>0$.



$mu$ is the separation constant .



I need to determine eigenvalues for each BVP involving $F$ and $G$. So for finding out eigenvalues i know i need to consider all the three cases
$mu>0$, $mu<0$ and $mu=0$ and then look for non-trivial solutions by applying the specific set of b.c. Although i am acquainted with the procedure to determine eigenvalues for a second order DE, the third order of the DE(s)is something i am not familiar with.



Any recommendations on how should i go about tackling this ?



Attempt
As per @Cesareo recommendations, I arrive at the following
linear equations



$$C_1+C_2+C_3=0$$



$$frac{F''(0)}{F'(0)}=frac{{C_1}{delta_1(mu)}^2+{C_2}{delta_2(mu)}^2+{C_3}{delta_3(mu)}^2}{-{C_1}{delta_1(mu)}-{C_2}{delta_2(mu)}-{C_3}{delta_3(mu)}}=beta_h$$



$$frac{F''(1)}{F'(1)}=frac{{C_1e^{-delta_1(mu)}}{delta_1(mu)}^2+{C_2e^{-delta_2(mu)}}{delta_2(mu)}^2+{C_3e^{-delta_3(mu)}}{delta_3(mu)}^2}{-{C_1e^{-delta_1(mu)}}{delta_1(mu)}-{C_2e^{-delta_2(mu)}}{delta_2(mu)}-{C_3e^{-delta_3(mu)}}{delta_3(mu)}}=beta_h$$



I reach the following form of $M(mu).C=0$



begin{vmatrix}
1 & 1 & 1 \
{delta_1(mu)}^2+beta_hdelta_1(mu) & {delta_2(mu)}^2+beta_hdelta_2(mu) & {delta_3(mu)}^2+beta_hdelta_3(mu) \
e^{-delta_1(mu)}({delta_1(mu)}^2+beta_hdelta_1(mu)) & e^{-delta_2(mu)}({delta_2(mu)}^2+beta_hdelta_2(mu)) & e^{-delta_3(mu)}({delta_3(mu)}^2+beta_hdelta_3(mu)) \
end{vmatrix}
$=0$



Solving this determinant is supposed to give me the eigen values $mu_n$ and consequently the eigen functions. The determinant can be reduced to two $0$ in the first row by coloumn manipulation, but I do not find any way to handle the consequent equation that comes out of it which is something like this:



$$[(delta_1(mu)-delta_2(mu))(delta_1(mu)+delta_2(mu)+beta_h)[(e^{-delta_2(mu)}{delta_2(mu)}^2-e^{-delta_3(mu)}{delta_3(mu)}^2)+beta_h(e^{-delta_2(mu)}{delta_2(mu)}-e^{-delta_3(mu)}{delta_3(mu)})]]-[(delta_2(mu)-delta_3(mu))(delta_2(mu)+delta_3(mu)+beta_h)[(e^{-delta_1(mu)}{delta_1(mu)}^2-e^{-delta_2(mu)}{delta_2(mu)}^2)+beta_h(e^{-delta_1(mu)}{delta_1(mu)}-e^{-delta_2(mu)}{delta_2(mu)})]]=0$$



After this step i fail to proceed further to find the eigenvalues using this $mathbb{det}M=0$ equation










share|cite|improve this question











$endgroup$

















    0












    $begingroup$


    I have two third order linear ODE which have been arrived after applying separation of variables to a system of PDEs



    begin{eqnarray}
    lambda_h F''' - 2 lambda_h beta_h F'' + left( (lambda_h beta_h - 1) beta_h - mu right) F' + beta_h^2 F &=& 0,\
    V lambda_c G''' - 2 V lambda_c beta_c G'' + left( (lambda_c beta_c - 1) V beta_c + mu right) G' + V beta_c^2 G &=& 0,
    end{eqnarray}



    $F$ is $F(x)$ and $G$ is $G(y)$. The boundary conditions are



    For $F$:
    $$F(0)=0$$
    $$frac{F''(0)}{F'(0)}=beta_h$$
    $$frac{F''(1)}{F'(1)}=beta_h$$



    For $G$:
    $$G(0)=0$$
    $$frac{G''(0)}{G'(0)}=beta_c$$
    $$frac{G''(1)}{G'(1)}=beta_c$$



    $lambda_h$, $lambda_c$, $beta_h$ and $beta_c$ are all constants $>0$.



    $mu$ is the separation constant .



    I need to determine eigenvalues for each BVP involving $F$ and $G$. So for finding out eigenvalues i know i need to consider all the three cases
    $mu>0$, $mu<0$ and $mu=0$ and then look for non-trivial solutions by applying the specific set of b.c. Although i am acquainted with the procedure to determine eigenvalues for a second order DE, the third order of the DE(s)is something i am not familiar with.



    Any recommendations on how should i go about tackling this ?



    Attempt
    As per @Cesareo recommendations, I arrive at the following
    linear equations



    $$C_1+C_2+C_3=0$$



    $$frac{F''(0)}{F'(0)}=frac{{C_1}{delta_1(mu)}^2+{C_2}{delta_2(mu)}^2+{C_3}{delta_3(mu)}^2}{-{C_1}{delta_1(mu)}-{C_2}{delta_2(mu)}-{C_3}{delta_3(mu)}}=beta_h$$



    $$frac{F''(1)}{F'(1)}=frac{{C_1e^{-delta_1(mu)}}{delta_1(mu)}^2+{C_2e^{-delta_2(mu)}}{delta_2(mu)}^2+{C_3e^{-delta_3(mu)}}{delta_3(mu)}^2}{-{C_1e^{-delta_1(mu)}}{delta_1(mu)}-{C_2e^{-delta_2(mu)}}{delta_2(mu)}-{C_3e^{-delta_3(mu)}}{delta_3(mu)}}=beta_h$$



    I reach the following form of $M(mu).C=0$



    begin{vmatrix}
    1 & 1 & 1 \
    {delta_1(mu)}^2+beta_hdelta_1(mu) & {delta_2(mu)}^2+beta_hdelta_2(mu) & {delta_3(mu)}^2+beta_hdelta_3(mu) \
    e^{-delta_1(mu)}({delta_1(mu)}^2+beta_hdelta_1(mu)) & e^{-delta_2(mu)}({delta_2(mu)}^2+beta_hdelta_2(mu)) & e^{-delta_3(mu)}({delta_3(mu)}^2+beta_hdelta_3(mu)) \
    end{vmatrix}
    $=0$



    Solving this determinant is supposed to give me the eigen values $mu_n$ and consequently the eigen functions. The determinant can be reduced to two $0$ in the first row by coloumn manipulation, but I do not find any way to handle the consequent equation that comes out of it which is something like this:



    $$[(delta_1(mu)-delta_2(mu))(delta_1(mu)+delta_2(mu)+beta_h)[(e^{-delta_2(mu)}{delta_2(mu)}^2-e^{-delta_3(mu)}{delta_3(mu)}^2)+beta_h(e^{-delta_2(mu)}{delta_2(mu)}-e^{-delta_3(mu)}{delta_3(mu)})]]-[(delta_2(mu)-delta_3(mu))(delta_2(mu)+delta_3(mu)+beta_h)[(e^{-delta_1(mu)}{delta_1(mu)}^2-e^{-delta_2(mu)}{delta_2(mu)}^2)+beta_h(e^{-delta_1(mu)}{delta_1(mu)}-e^{-delta_2(mu)}{delta_2(mu)})]]=0$$



    After this step i fail to proceed further to find the eigenvalues using this $mathbb{det}M=0$ equation










    share|cite|improve this question











    $endgroup$















      0












      0








      0





      $begingroup$


      I have two third order linear ODE which have been arrived after applying separation of variables to a system of PDEs



      begin{eqnarray}
      lambda_h F''' - 2 lambda_h beta_h F'' + left( (lambda_h beta_h - 1) beta_h - mu right) F' + beta_h^2 F &=& 0,\
      V lambda_c G''' - 2 V lambda_c beta_c G'' + left( (lambda_c beta_c - 1) V beta_c + mu right) G' + V beta_c^2 G &=& 0,
      end{eqnarray}



      $F$ is $F(x)$ and $G$ is $G(y)$. The boundary conditions are



      For $F$:
      $$F(0)=0$$
      $$frac{F''(0)}{F'(0)}=beta_h$$
      $$frac{F''(1)}{F'(1)}=beta_h$$



      For $G$:
      $$G(0)=0$$
      $$frac{G''(0)}{G'(0)}=beta_c$$
      $$frac{G''(1)}{G'(1)}=beta_c$$



      $lambda_h$, $lambda_c$, $beta_h$ and $beta_c$ are all constants $>0$.



      $mu$ is the separation constant .



      I need to determine eigenvalues for each BVP involving $F$ and $G$. So for finding out eigenvalues i know i need to consider all the three cases
      $mu>0$, $mu<0$ and $mu=0$ and then look for non-trivial solutions by applying the specific set of b.c. Although i am acquainted with the procedure to determine eigenvalues for a second order DE, the third order of the DE(s)is something i am not familiar with.



      Any recommendations on how should i go about tackling this ?



      Attempt
      As per @Cesareo recommendations, I arrive at the following
      linear equations



      $$C_1+C_2+C_3=0$$



      $$frac{F''(0)}{F'(0)}=frac{{C_1}{delta_1(mu)}^2+{C_2}{delta_2(mu)}^2+{C_3}{delta_3(mu)}^2}{-{C_1}{delta_1(mu)}-{C_2}{delta_2(mu)}-{C_3}{delta_3(mu)}}=beta_h$$



      $$frac{F''(1)}{F'(1)}=frac{{C_1e^{-delta_1(mu)}}{delta_1(mu)}^2+{C_2e^{-delta_2(mu)}}{delta_2(mu)}^2+{C_3e^{-delta_3(mu)}}{delta_3(mu)}^2}{-{C_1e^{-delta_1(mu)}}{delta_1(mu)}-{C_2e^{-delta_2(mu)}}{delta_2(mu)}-{C_3e^{-delta_3(mu)}}{delta_3(mu)}}=beta_h$$



      I reach the following form of $M(mu).C=0$



      begin{vmatrix}
      1 & 1 & 1 \
      {delta_1(mu)}^2+beta_hdelta_1(mu) & {delta_2(mu)}^2+beta_hdelta_2(mu) & {delta_3(mu)}^2+beta_hdelta_3(mu) \
      e^{-delta_1(mu)}({delta_1(mu)}^2+beta_hdelta_1(mu)) & e^{-delta_2(mu)}({delta_2(mu)}^2+beta_hdelta_2(mu)) & e^{-delta_3(mu)}({delta_3(mu)}^2+beta_hdelta_3(mu)) \
      end{vmatrix}
      $=0$



      Solving this determinant is supposed to give me the eigen values $mu_n$ and consequently the eigen functions. The determinant can be reduced to two $0$ in the first row by coloumn manipulation, but I do not find any way to handle the consequent equation that comes out of it which is something like this:



      $$[(delta_1(mu)-delta_2(mu))(delta_1(mu)+delta_2(mu)+beta_h)[(e^{-delta_2(mu)}{delta_2(mu)}^2-e^{-delta_3(mu)}{delta_3(mu)}^2)+beta_h(e^{-delta_2(mu)}{delta_2(mu)}-e^{-delta_3(mu)}{delta_3(mu)})]]-[(delta_2(mu)-delta_3(mu))(delta_2(mu)+delta_3(mu)+beta_h)[(e^{-delta_1(mu)}{delta_1(mu)}^2-e^{-delta_2(mu)}{delta_2(mu)}^2)+beta_h(e^{-delta_1(mu)}{delta_1(mu)}-e^{-delta_2(mu)}{delta_2(mu)})]]=0$$



      After this step i fail to proceed further to find the eigenvalues using this $mathbb{det}M=0$ equation










      share|cite|improve this question











      $endgroup$




      I have two third order linear ODE which have been arrived after applying separation of variables to a system of PDEs



      begin{eqnarray}
      lambda_h F''' - 2 lambda_h beta_h F'' + left( (lambda_h beta_h - 1) beta_h - mu right) F' + beta_h^2 F &=& 0,\
      V lambda_c G''' - 2 V lambda_c beta_c G'' + left( (lambda_c beta_c - 1) V beta_c + mu right) G' + V beta_c^2 G &=& 0,
      end{eqnarray}



      $F$ is $F(x)$ and $G$ is $G(y)$. The boundary conditions are



      For $F$:
      $$F(0)=0$$
      $$frac{F''(0)}{F'(0)}=beta_h$$
      $$frac{F''(1)}{F'(1)}=beta_h$$



      For $G$:
      $$G(0)=0$$
      $$frac{G''(0)}{G'(0)}=beta_c$$
      $$frac{G''(1)}{G'(1)}=beta_c$$



      $lambda_h$, $lambda_c$, $beta_h$ and $beta_c$ are all constants $>0$.



      $mu$ is the separation constant .



      I need to determine eigenvalues for each BVP involving $F$ and $G$. So for finding out eigenvalues i know i need to consider all the three cases
      $mu>0$, $mu<0$ and $mu=0$ and then look for non-trivial solutions by applying the specific set of b.c. Although i am acquainted with the procedure to determine eigenvalues for a second order DE, the third order of the DE(s)is something i am not familiar with.



      Any recommendations on how should i go about tackling this ?



      Attempt
      As per @Cesareo recommendations, I arrive at the following
      linear equations



      $$C_1+C_2+C_3=0$$



      $$frac{F''(0)}{F'(0)}=frac{{C_1}{delta_1(mu)}^2+{C_2}{delta_2(mu)}^2+{C_3}{delta_3(mu)}^2}{-{C_1}{delta_1(mu)}-{C_2}{delta_2(mu)}-{C_3}{delta_3(mu)}}=beta_h$$



      $$frac{F''(1)}{F'(1)}=frac{{C_1e^{-delta_1(mu)}}{delta_1(mu)}^2+{C_2e^{-delta_2(mu)}}{delta_2(mu)}^2+{C_3e^{-delta_3(mu)}}{delta_3(mu)}^2}{-{C_1e^{-delta_1(mu)}}{delta_1(mu)}-{C_2e^{-delta_2(mu)}}{delta_2(mu)}-{C_3e^{-delta_3(mu)}}{delta_3(mu)}}=beta_h$$



      I reach the following form of $M(mu).C=0$



      begin{vmatrix}
      1 & 1 & 1 \
      {delta_1(mu)}^2+beta_hdelta_1(mu) & {delta_2(mu)}^2+beta_hdelta_2(mu) & {delta_3(mu)}^2+beta_hdelta_3(mu) \
      e^{-delta_1(mu)}({delta_1(mu)}^2+beta_hdelta_1(mu)) & e^{-delta_2(mu)}({delta_2(mu)}^2+beta_hdelta_2(mu)) & e^{-delta_3(mu)}({delta_3(mu)}^2+beta_hdelta_3(mu)) \
      end{vmatrix}
      $=0$



      Solving this determinant is supposed to give me the eigen values $mu_n$ and consequently the eigen functions. The determinant can be reduced to two $0$ in the first row by coloumn manipulation, but I do not find any way to handle the consequent equation that comes out of it which is something like this:



      $$[(delta_1(mu)-delta_2(mu))(delta_1(mu)+delta_2(mu)+beta_h)[(e^{-delta_2(mu)}{delta_2(mu)}^2-e^{-delta_3(mu)}{delta_3(mu)}^2)+beta_h(e^{-delta_2(mu)}{delta_2(mu)}-e^{-delta_3(mu)}{delta_3(mu)})]]-[(delta_2(mu)-delta_3(mu))(delta_2(mu)+delta_3(mu)+beta_h)[(e^{-delta_1(mu)}{delta_1(mu)}^2-e^{-delta_2(mu)}{delta_2(mu)}^2)+beta_h(e^{-delta_1(mu)}{delta_1(mu)}-e^{-delta_2(mu)}{delta_2(mu)})]]=0$$



      After this step i fail to proceed further to find the eigenvalues using this $mathbb{det}M=0$ equation







      ordinary-differential-equations pde eigenvalues-eigenvectors eigenfunctions






      share|cite|improve this question















      share|cite|improve this question













      share|cite|improve this question




      share|cite|improve this question








      edited Jan 13 at 3:42







      Indrasis Mitra

















      asked Jan 11 at 12:06









      Indrasis MitraIndrasis Mitra

      2517




      2517






















          1 Answer
          1






          active

          oldest

          votes


















          1












          $begingroup$

          Regarding the first DE the linear differential operator



          $$
          lambda_h delta^3 - 2 lambda_h beta_h delta^2 + left( (lambda_h beta_h - 1) beta_h - mu right) delta + beta_h^2=0
          $$



          and the three roots $delta_i(mu), i = 1,2,3$ we have that



          $$
          F(t) = sum_k C_k e^{-delta_k(mu)t}
          $$



          using now the boundary conditions



          $$
          F(0) = sum_k C_k = 0 longrightarrow (1)
          $$



          and with



          $$
          F'(0) = -sum_k C_k delta_k(mu)\
          F''(0) = sum_k C_k delta_k(mu)^2\
          $$



          giving



          $$
          -frac{sum_k C_k delta_k(mu)^2}{sum_k C_k delta_k(mu)}=beta_hlongrightarrow (2)
          $$



          and similarly



          $$
          -frac{sum_k C_k delta_k(mu)^2e^{-delta_k(mu)}}{sum_k C_k delta_k(mu)e^{-delta_k(mu)}}=beta_hlongrightarrow (3)
          $$



          then we have three linear equations $(1,2,3)$ in $C_k$ that can be arranged as



          $$
          M(mu)cdot C = 0, C = (C_k)
          $$



          This system have nontrivial solution for $det(M(mu)) = 0$ hence the roots for this determinant equation are the eigenvalues $mu_n$ and the eigenfunctions are $e^{-delta_k(mu_n)t}$



          The procedure for $G$ is quite similar.



          NOTE



          Assuming numerical values $lambda_h = 1,beta_h = -10$ we have the operator polynomial



          $$
          s^3+20s^2+(110-mu)s+100 = 0
          $$



          with roots $delta_1(mu),delta_2(mu),delta_3(mu)$



          The determinant after simplifications reads



          $$
          det(M) = left(e^{delta _1+delta _2} left(delta _1-delta _2right) delta _3 left(beta
          _h+delta _1+delta _2right) left(beta _h+delta _3right)-e^{delta _1+delta _3}
          delta _2 left(delta _1-delta _3right) left(beta _h+delta _2right) left(beta
          _h+delta _1+delta _3right)+e^{delta _2+delta _3} delta _1 left(delta _2-delta
          _3right) left(beta _h+delta _1right) left(beta _h+delta _2+delta
          _3right)right) left(cosh left(delta _1+delta _2+delta _3right)-sinh
          left(delta _1+delta _2+delta _3right)right)
          $$



          discarding $cosh (delta_1+delta_2+delta_3)-sinh
          (delta_1+delta_2+delta_3)=0$
          we follow with



          $$
          Delta(mu)=e^{delta _1+delta _2} left(delta _1-delta _2right) delta _3 left(beta _h+delta
          _1+delta _2right) left(beta _h+delta _3right)-e^{delta _1+delta _3} delta _2
          left(delta _1-delta _3right) left(beta _h+delta _2right) left(beta _h+delta
          _1+delta _3right)+e^{delta _2+delta _3} delta _1 left(delta _2-delta _3right)
          left(beta _h+delta _1right) left(beta _h+delta _2+delta _3right)=0
          $$



          and then after plotting we have



          enter image description here



          In red Re[$Delta(mu)$] and in blue Im[$Delta(mu)$]. The zeroes are the eigenvalues $mu_n$



          Attached a very basic MATHEMATICA script in order to obtain the first $mu_k$ for $lambda_h = frac 14, beta_h = -10$



          parms = {lh -> 1/4, bh -> -10};
          F[t_, n_] := Sum[
          !(*SubscriptBox[(c), ({1, j})]) Exp[exps[[j]][[1]] t] +
          !(*SubscriptBox[(c), ({2, j})]) Exp[exps[[j]][[2]] t] +
          !(*SubscriptBox[(c), ({3, j})]) Exp[exps[[j]][[3]] t], {j, 1,n}]
          sols = Solve[
          lh s^3 - 2 lh bh s^2 + ((lh bh - 1) bh - mu) s + bh^2 == 0, s] /.
          parms // FullSimplify
          roots = s /. sols;
          M = {{1, 1, 1}, {r1^2 + bh r1, r2^2 + bh r2, r3^2 + bh r3},
          {E^(-r1) (r1^2 + bh r1), E^(-r2) (r2^2 + bh r2), E^(-r3) (r3^2 + bh r3)}};
          det = -Det[M] // FullSimplify
          subdet1 = (Cosh[r1 + r2 + r3] - Sinh[r1 + r2 + r3])
          subdet2 = det/subdet1 // FullSimplify
          subdet20 = subdet2 /. Thread[{r1, r2, r3} -> roots] /. parms;
          Plot[Im[subdet20], {mu, -100, 0}, PlotStyle -> {Thick, Black},
          PlotRange -> {-10, 10}]
          solmu1 = FindRoot[Im[subdet20] == 0, {mu, 0}]
          solmu2 = FindRoot[Im[subdet20] == 0, {mu, -7}]
          solmu3 = FindRoot[Im[subdet20] == 0, {mu, -20}]
          solmu4 = FindRoot[Im[subdet20] == 0, {mu, -40}]
          solmu5 = FindRoot[Im[subdet20] == 0, {mu, -60}]
          solmu6 = FindRoot[Im[subdet20] == 0, {mu, -80}]
          solmu7 = FindRoot[Im[subdet20] == 0, {mu, -110}]
          solmu8 = FindRoot[Im[subdet20] == 0, {mu, -150}]
          solmu9 = FindRoot[Im[subdet20] == 0, {mu, -190}]
          Subscript[mu, 1] = mu /. solmu1;
          Subscript[mu, 2] = mu /. solmu2;
          Subscript[mu, 3] = mu /. solmu3;
          Subscript[mu, 4] = mu /. solmu4;
          Subscript[mu, 5] = mu /. solmu5;
          Subscript[mu, 6] = mu /. solmu6;
          Subscript[mu, 7] = mu /. solmu7;
          Subscript[mu, 8] = mu /. solmu8;
          Subscript[mu, 9] = mu /. solmu9;
          exps = Table[roots /. parms /. {mu -> Subscript[mu, k]}, {k, 1, 9}]
          F[t, 9]





          share|cite|improve this answer











          $endgroup$













          • $begingroup$
            Thanks for the explanation. I guess you used $F'/F''$ while writing $(2)$ and $(3)$ while the BC are actually $F''/F'$
            $endgroup$
            – Indrasis Mitra
            Jan 12 at 3:10










          • $begingroup$
            Also will solving for $det(M(mu))=0$ take care of all the possible values of $mu$ that needs to be considered i.e. $mu>0$,$mu=0$ and $mu<0$ ? for finding the eigenvalues ?
            $endgroup$
            – Indrasis Mitra
            Jan 12 at 4:26










          • $begingroup$
            my last comment is pretty ignorant. So actually $F(t)=C_1e^{-delta_1(mu)t}+C_2e^{-delta_2(mu)t}+C_3e^{-delta_3(mu)t}$, for the three roots of the characteristic equation. Am i right ?
            $endgroup$
            – Indrasis Mitra
            Jan 12 at 6:40












          • $begingroup$
            I followed the steps you suggested to atrrive at $mathbb{det}(M(mu))=0$. I find that it has $delta_1(mu)$,$delta_2(mu)$ and $delta_3(mu)$. I have edited the original question to reflect my attempt. I cannot figure out how to proceed further, am i doing something wrong ?
            $endgroup$
            – Indrasis Mitra
            Jan 13 at 3:45










          • $begingroup$
            @IndrasisMitra See attached note.
            $endgroup$
            – Cesareo
            Jan 13 at 9:19











          Your Answer





          StackExchange.ifUsing("editor", function () {
          return StackExchange.using("mathjaxEditing", function () {
          StackExchange.MarkdownEditor.creationCallbacks.add(function (editor, postfix) {
          StackExchange.mathjaxEditing.prepareWmdForMathJax(editor, postfix, [["$", "$"], ["\\(","\\)"]]);
          });
          });
          }, "mathjax-editing");

          StackExchange.ready(function() {
          var channelOptions = {
          tags: "".split(" "),
          id: "69"
          };
          initTagRenderer("".split(" "), "".split(" "), channelOptions);

          StackExchange.using("externalEditor", function() {
          // Have to fire editor after snippets, if snippets enabled
          if (StackExchange.settings.snippets.snippetsEnabled) {
          StackExchange.using("snippets", function() {
          createEditor();
          });
          }
          else {
          createEditor();
          }
          });

          function createEditor() {
          StackExchange.prepareEditor({
          heartbeatType: 'answer',
          autoActivateHeartbeat: false,
          convertImagesToLinks: true,
          noModals: true,
          showLowRepImageUploadWarning: true,
          reputationToPostImages: 10,
          bindNavPrevention: true,
          postfix: "",
          imageUploader: {
          brandingHtml: "Powered by u003ca class="icon-imgur-white" href="https://imgur.com/"u003eu003c/au003e",
          contentPolicyHtml: "User contributions licensed under u003ca href="https://creativecommons.org/licenses/by-sa/3.0/"u003ecc by-sa 3.0 with attribution requiredu003c/au003e u003ca href="https://stackoverflow.com/legal/content-policy"u003e(content policy)u003c/au003e",
          allowUrls: true
          },
          noCode: true, onDemand: true,
          discardSelector: ".discard-answer"
          ,immediatelyShowMarkdownHelp:true
          });


          }
          });














          draft saved

          draft discarded


















          StackExchange.ready(
          function () {
          StackExchange.openid.initPostLogin('.new-post-login', 'https%3a%2f%2fmath.stackexchange.com%2fquestions%2f3069768%2feigen-values-of-a-third-order-linear-homogenous-ode%23new-answer', 'question_page');
          }
          );

          Post as a guest















          Required, but never shown

























          1 Answer
          1






          active

          oldest

          votes








          1 Answer
          1






          active

          oldest

          votes









          active

          oldest

          votes






          active

          oldest

          votes









          1












          $begingroup$

          Regarding the first DE the linear differential operator



          $$
          lambda_h delta^3 - 2 lambda_h beta_h delta^2 + left( (lambda_h beta_h - 1) beta_h - mu right) delta + beta_h^2=0
          $$



          and the three roots $delta_i(mu), i = 1,2,3$ we have that



          $$
          F(t) = sum_k C_k e^{-delta_k(mu)t}
          $$



          using now the boundary conditions



          $$
          F(0) = sum_k C_k = 0 longrightarrow (1)
          $$



          and with



          $$
          F'(0) = -sum_k C_k delta_k(mu)\
          F''(0) = sum_k C_k delta_k(mu)^2\
          $$



          giving



          $$
          -frac{sum_k C_k delta_k(mu)^2}{sum_k C_k delta_k(mu)}=beta_hlongrightarrow (2)
          $$



          and similarly



          $$
          -frac{sum_k C_k delta_k(mu)^2e^{-delta_k(mu)}}{sum_k C_k delta_k(mu)e^{-delta_k(mu)}}=beta_hlongrightarrow (3)
          $$



          then we have three linear equations $(1,2,3)$ in $C_k$ that can be arranged as



          $$
          M(mu)cdot C = 0, C = (C_k)
          $$



          This system have nontrivial solution for $det(M(mu)) = 0$ hence the roots for this determinant equation are the eigenvalues $mu_n$ and the eigenfunctions are $e^{-delta_k(mu_n)t}$



          The procedure for $G$ is quite similar.



          NOTE



          Assuming numerical values $lambda_h = 1,beta_h = -10$ we have the operator polynomial



          $$
          s^3+20s^2+(110-mu)s+100 = 0
          $$



          with roots $delta_1(mu),delta_2(mu),delta_3(mu)$



          The determinant after simplifications reads



          $$
          det(M) = left(e^{delta _1+delta _2} left(delta _1-delta _2right) delta _3 left(beta
          _h+delta _1+delta _2right) left(beta _h+delta _3right)-e^{delta _1+delta _3}
          delta _2 left(delta _1-delta _3right) left(beta _h+delta _2right) left(beta
          _h+delta _1+delta _3right)+e^{delta _2+delta _3} delta _1 left(delta _2-delta
          _3right) left(beta _h+delta _1right) left(beta _h+delta _2+delta
          _3right)right) left(cosh left(delta _1+delta _2+delta _3right)-sinh
          left(delta _1+delta _2+delta _3right)right)
          $$



          discarding $cosh (delta_1+delta_2+delta_3)-sinh
          (delta_1+delta_2+delta_3)=0$
          we follow with



          $$
          Delta(mu)=e^{delta _1+delta _2} left(delta _1-delta _2right) delta _3 left(beta _h+delta
          _1+delta _2right) left(beta _h+delta _3right)-e^{delta _1+delta _3} delta _2
          left(delta _1-delta _3right) left(beta _h+delta _2right) left(beta _h+delta
          _1+delta _3right)+e^{delta _2+delta _3} delta _1 left(delta _2-delta _3right)
          left(beta _h+delta _1right) left(beta _h+delta _2+delta _3right)=0
          $$



          and then after plotting we have



          enter image description here



          In red Re[$Delta(mu)$] and in blue Im[$Delta(mu)$]. The zeroes are the eigenvalues $mu_n$



          Attached a very basic MATHEMATICA script in order to obtain the first $mu_k$ for $lambda_h = frac 14, beta_h = -10$



          parms = {lh -> 1/4, bh -> -10};
          F[t_, n_] := Sum[
          !(*SubscriptBox[(c), ({1, j})]) Exp[exps[[j]][[1]] t] +
          !(*SubscriptBox[(c), ({2, j})]) Exp[exps[[j]][[2]] t] +
          !(*SubscriptBox[(c), ({3, j})]) Exp[exps[[j]][[3]] t], {j, 1,n}]
          sols = Solve[
          lh s^3 - 2 lh bh s^2 + ((lh bh - 1) bh - mu) s + bh^2 == 0, s] /.
          parms // FullSimplify
          roots = s /. sols;
          M = {{1, 1, 1}, {r1^2 + bh r1, r2^2 + bh r2, r3^2 + bh r3},
          {E^(-r1) (r1^2 + bh r1), E^(-r2) (r2^2 + bh r2), E^(-r3) (r3^2 + bh r3)}};
          det = -Det[M] // FullSimplify
          subdet1 = (Cosh[r1 + r2 + r3] - Sinh[r1 + r2 + r3])
          subdet2 = det/subdet1 // FullSimplify
          subdet20 = subdet2 /. Thread[{r1, r2, r3} -> roots] /. parms;
          Plot[Im[subdet20], {mu, -100, 0}, PlotStyle -> {Thick, Black},
          PlotRange -> {-10, 10}]
          solmu1 = FindRoot[Im[subdet20] == 0, {mu, 0}]
          solmu2 = FindRoot[Im[subdet20] == 0, {mu, -7}]
          solmu3 = FindRoot[Im[subdet20] == 0, {mu, -20}]
          solmu4 = FindRoot[Im[subdet20] == 0, {mu, -40}]
          solmu5 = FindRoot[Im[subdet20] == 0, {mu, -60}]
          solmu6 = FindRoot[Im[subdet20] == 0, {mu, -80}]
          solmu7 = FindRoot[Im[subdet20] == 0, {mu, -110}]
          solmu8 = FindRoot[Im[subdet20] == 0, {mu, -150}]
          solmu9 = FindRoot[Im[subdet20] == 0, {mu, -190}]
          Subscript[mu, 1] = mu /. solmu1;
          Subscript[mu, 2] = mu /. solmu2;
          Subscript[mu, 3] = mu /. solmu3;
          Subscript[mu, 4] = mu /. solmu4;
          Subscript[mu, 5] = mu /. solmu5;
          Subscript[mu, 6] = mu /. solmu6;
          Subscript[mu, 7] = mu /. solmu7;
          Subscript[mu, 8] = mu /. solmu8;
          Subscript[mu, 9] = mu /. solmu9;
          exps = Table[roots /. parms /. {mu -> Subscript[mu, k]}, {k, 1, 9}]
          F[t, 9]





          share|cite|improve this answer











          $endgroup$













          • $begingroup$
            Thanks for the explanation. I guess you used $F'/F''$ while writing $(2)$ and $(3)$ while the BC are actually $F''/F'$
            $endgroup$
            – Indrasis Mitra
            Jan 12 at 3:10










          • $begingroup$
            Also will solving for $det(M(mu))=0$ take care of all the possible values of $mu$ that needs to be considered i.e. $mu>0$,$mu=0$ and $mu<0$ ? for finding the eigenvalues ?
            $endgroup$
            – Indrasis Mitra
            Jan 12 at 4:26










          • $begingroup$
            my last comment is pretty ignorant. So actually $F(t)=C_1e^{-delta_1(mu)t}+C_2e^{-delta_2(mu)t}+C_3e^{-delta_3(mu)t}$, for the three roots of the characteristic equation. Am i right ?
            $endgroup$
            – Indrasis Mitra
            Jan 12 at 6:40












          • $begingroup$
            I followed the steps you suggested to atrrive at $mathbb{det}(M(mu))=0$. I find that it has $delta_1(mu)$,$delta_2(mu)$ and $delta_3(mu)$. I have edited the original question to reflect my attempt. I cannot figure out how to proceed further, am i doing something wrong ?
            $endgroup$
            – Indrasis Mitra
            Jan 13 at 3:45










          • $begingroup$
            @IndrasisMitra See attached note.
            $endgroup$
            – Cesareo
            Jan 13 at 9:19
















          1












          $begingroup$

          Regarding the first DE the linear differential operator



          $$
          lambda_h delta^3 - 2 lambda_h beta_h delta^2 + left( (lambda_h beta_h - 1) beta_h - mu right) delta + beta_h^2=0
          $$



          and the three roots $delta_i(mu), i = 1,2,3$ we have that



          $$
          F(t) = sum_k C_k e^{-delta_k(mu)t}
          $$



          using now the boundary conditions



          $$
          F(0) = sum_k C_k = 0 longrightarrow (1)
          $$



          and with



          $$
          F'(0) = -sum_k C_k delta_k(mu)\
          F''(0) = sum_k C_k delta_k(mu)^2\
          $$



          giving



          $$
          -frac{sum_k C_k delta_k(mu)^2}{sum_k C_k delta_k(mu)}=beta_hlongrightarrow (2)
          $$



          and similarly



          $$
          -frac{sum_k C_k delta_k(mu)^2e^{-delta_k(mu)}}{sum_k C_k delta_k(mu)e^{-delta_k(mu)}}=beta_hlongrightarrow (3)
          $$



          then we have three linear equations $(1,2,3)$ in $C_k$ that can be arranged as



          $$
          M(mu)cdot C = 0, C = (C_k)
          $$



          This system have nontrivial solution for $det(M(mu)) = 0$ hence the roots for this determinant equation are the eigenvalues $mu_n$ and the eigenfunctions are $e^{-delta_k(mu_n)t}$



          The procedure for $G$ is quite similar.



          NOTE



          Assuming numerical values $lambda_h = 1,beta_h = -10$ we have the operator polynomial



          $$
          s^3+20s^2+(110-mu)s+100 = 0
          $$



          with roots $delta_1(mu),delta_2(mu),delta_3(mu)$



          The determinant after simplifications reads



          $$
          det(M) = left(e^{delta _1+delta _2} left(delta _1-delta _2right) delta _3 left(beta
          _h+delta _1+delta _2right) left(beta _h+delta _3right)-e^{delta _1+delta _3}
          delta _2 left(delta _1-delta _3right) left(beta _h+delta _2right) left(beta
          _h+delta _1+delta _3right)+e^{delta _2+delta _3} delta _1 left(delta _2-delta
          _3right) left(beta _h+delta _1right) left(beta _h+delta _2+delta
          _3right)right) left(cosh left(delta _1+delta _2+delta _3right)-sinh
          left(delta _1+delta _2+delta _3right)right)
          $$



          discarding $cosh (delta_1+delta_2+delta_3)-sinh
          (delta_1+delta_2+delta_3)=0$
          we follow with



          $$
          Delta(mu)=e^{delta _1+delta _2} left(delta _1-delta _2right) delta _3 left(beta _h+delta
          _1+delta _2right) left(beta _h+delta _3right)-e^{delta _1+delta _3} delta _2
          left(delta _1-delta _3right) left(beta _h+delta _2right) left(beta _h+delta
          _1+delta _3right)+e^{delta _2+delta _3} delta _1 left(delta _2-delta _3right)
          left(beta _h+delta _1right) left(beta _h+delta _2+delta _3right)=0
          $$



          and then after plotting we have



          enter image description here



          In red Re[$Delta(mu)$] and in blue Im[$Delta(mu)$]. The zeroes are the eigenvalues $mu_n$



          Attached a very basic MATHEMATICA script in order to obtain the first $mu_k$ for $lambda_h = frac 14, beta_h = -10$



          parms = {lh -> 1/4, bh -> -10};
          F[t_, n_] := Sum[
          !(*SubscriptBox[(c), ({1, j})]) Exp[exps[[j]][[1]] t] +
          !(*SubscriptBox[(c), ({2, j})]) Exp[exps[[j]][[2]] t] +
          !(*SubscriptBox[(c), ({3, j})]) Exp[exps[[j]][[3]] t], {j, 1,n}]
          sols = Solve[
          lh s^3 - 2 lh bh s^2 + ((lh bh - 1) bh - mu) s + bh^2 == 0, s] /.
          parms // FullSimplify
          roots = s /. sols;
          M = {{1, 1, 1}, {r1^2 + bh r1, r2^2 + bh r2, r3^2 + bh r3},
          {E^(-r1) (r1^2 + bh r1), E^(-r2) (r2^2 + bh r2), E^(-r3) (r3^2 + bh r3)}};
          det = -Det[M] // FullSimplify
          subdet1 = (Cosh[r1 + r2 + r3] - Sinh[r1 + r2 + r3])
          subdet2 = det/subdet1 // FullSimplify
          subdet20 = subdet2 /. Thread[{r1, r2, r3} -> roots] /. parms;
          Plot[Im[subdet20], {mu, -100, 0}, PlotStyle -> {Thick, Black},
          PlotRange -> {-10, 10}]
          solmu1 = FindRoot[Im[subdet20] == 0, {mu, 0}]
          solmu2 = FindRoot[Im[subdet20] == 0, {mu, -7}]
          solmu3 = FindRoot[Im[subdet20] == 0, {mu, -20}]
          solmu4 = FindRoot[Im[subdet20] == 0, {mu, -40}]
          solmu5 = FindRoot[Im[subdet20] == 0, {mu, -60}]
          solmu6 = FindRoot[Im[subdet20] == 0, {mu, -80}]
          solmu7 = FindRoot[Im[subdet20] == 0, {mu, -110}]
          solmu8 = FindRoot[Im[subdet20] == 0, {mu, -150}]
          solmu9 = FindRoot[Im[subdet20] == 0, {mu, -190}]
          Subscript[mu, 1] = mu /. solmu1;
          Subscript[mu, 2] = mu /. solmu2;
          Subscript[mu, 3] = mu /. solmu3;
          Subscript[mu, 4] = mu /. solmu4;
          Subscript[mu, 5] = mu /. solmu5;
          Subscript[mu, 6] = mu /. solmu6;
          Subscript[mu, 7] = mu /. solmu7;
          Subscript[mu, 8] = mu /. solmu8;
          Subscript[mu, 9] = mu /. solmu9;
          exps = Table[roots /. parms /. {mu -> Subscript[mu, k]}, {k, 1, 9}]
          F[t, 9]





          share|cite|improve this answer











          $endgroup$













          • $begingroup$
            Thanks for the explanation. I guess you used $F'/F''$ while writing $(2)$ and $(3)$ while the BC are actually $F''/F'$
            $endgroup$
            – Indrasis Mitra
            Jan 12 at 3:10










          • $begingroup$
            Also will solving for $det(M(mu))=0$ take care of all the possible values of $mu$ that needs to be considered i.e. $mu>0$,$mu=0$ and $mu<0$ ? for finding the eigenvalues ?
            $endgroup$
            – Indrasis Mitra
            Jan 12 at 4:26










          • $begingroup$
            my last comment is pretty ignorant. So actually $F(t)=C_1e^{-delta_1(mu)t}+C_2e^{-delta_2(mu)t}+C_3e^{-delta_3(mu)t}$, for the three roots of the characteristic equation. Am i right ?
            $endgroup$
            – Indrasis Mitra
            Jan 12 at 6:40












          • $begingroup$
            I followed the steps you suggested to atrrive at $mathbb{det}(M(mu))=0$. I find that it has $delta_1(mu)$,$delta_2(mu)$ and $delta_3(mu)$. I have edited the original question to reflect my attempt. I cannot figure out how to proceed further, am i doing something wrong ?
            $endgroup$
            – Indrasis Mitra
            Jan 13 at 3:45










          • $begingroup$
            @IndrasisMitra See attached note.
            $endgroup$
            – Cesareo
            Jan 13 at 9:19














          1












          1








          1





          $begingroup$

          Regarding the first DE the linear differential operator



          $$
          lambda_h delta^3 - 2 lambda_h beta_h delta^2 + left( (lambda_h beta_h - 1) beta_h - mu right) delta + beta_h^2=0
          $$



          and the three roots $delta_i(mu), i = 1,2,3$ we have that



          $$
          F(t) = sum_k C_k e^{-delta_k(mu)t}
          $$



          using now the boundary conditions



          $$
          F(0) = sum_k C_k = 0 longrightarrow (1)
          $$



          and with



          $$
          F'(0) = -sum_k C_k delta_k(mu)\
          F''(0) = sum_k C_k delta_k(mu)^2\
          $$



          giving



          $$
          -frac{sum_k C_k delta_k(mu)^2}{sum_k C_k delta_k(mu)}=beta_hlongrightarrow (2)
          $$



          and similarly



          $$
          -frac{sum_k C_k delta_k(mu)^2e^{-delta_k(mu)}}{sum_k C_k delta_k(mu)e^{-delta_k(mu)}}=beta_hlongrightarrow (3)
          $$



          then we have three linear equations $(1,2,3)$ in $C_k$ that can be arranged as



          $$
          M(mu)cdot C = 0, C = (C_k)
          $$



          This system have nontrivial solution for $det(M(mu)) = 0$ hence the roots for this determinant equation are the eigenvalues $mu_n$ and the eigenfunctions are $e^{-delta_k(mu_n)t}$



          The procedure for $G$ is quite similar.



          NOTE



          Assuming numerical values $lambda_h = 1,beta_h = -10$ we have the operator polynomial



          $$
          s^3+20s^2+(110-mu)s+100 = 0
          $$



          with roots $delta_1(mu),delta_2(mu),delta_3(mu)$



          The determinant after simplifications reads



          $$
          det(M) = left(e^{delta _1+delta _2} left(delta _1-delta _2right) delta _3 left(beta
          _h+delta _1+delta _2right) left(beta _h+delta _3right)-e^{delta _1+delta _3}
          delta _2 left(delta _1-delta _3right) left(beta _h+delta _2right) left(beta
          _h+delta _1+delta _3right)+e^{delta _2+delta _3} delta _1 left(delta _2-delta
          _3right) left(beta _h+delta _1right) left(beta _h+delta _2+delta
          _3right)right) left(cosh left(delta _1+delta _2+delta _3right)-sinh
          left(delta _1+delta _2+delta _3right)right)
          $$



          discarding $cosh (delta_1+delta_2+delta_3)-sinh
          (delta_1+delta_2+delta_3)=0$
          we follow with



          $$
          Delta(mu)=e^{delta _1+delta _2} left(delta _1-delta _2right) delta _3 left(beta _h+delta
          _1+delta _2right) left(beta _h+delta _3right)-e^{delta _1+delta _3} delta _2
          left(delta _1-delta _3right) left(beta _h+delta _2right) left(beta _h+delta
          _1+delta _3right)+e^{delta _2+delta _3} delta _1 left(delta _2-delta _3right)
          left(beta _h+delta _1right) left(beta _h+delta _2+delta _3right)=0
          $$



          and then after plotting we have



          enter image description here



          In red Re[$Delta(mu)$] and in blue Im[$Delta(mu)$]. The zeroes are the eigenvalues $mu_n$



          Attached a very basic MATHEMATICA script in order to obtain the first $mu_k$ for $lambda_h = frac 14, beta_h = -10$



          parms = {lh -> 1/4, bh -> -10};
          F[t_, n_] := Sum[
          !(*SubscriptBox[(c), ({1, j})]) Exp[exps[[j]][[1]] t] +
          !(*SubscriptBox[(c), ({2, j})]) Exp[exps[[j]][[2]] t] +
          !(*SubscriptBox[(c), ({3, j})]) Exp[exps[[j]][[3]] t], {j, 1,n}]
          sols = Solve[
          lh s^3 - 2 lh bh s^2 + ((lh bh - 1) bh - mu) s + bh^2 == 0, s] /.
          parms // FullSimplify
          roots = s /. sols;
          M = {{1, 1, 1}, {r1^2 + bh r1, r2^2 + bh r2, r3^2 + bh r3},
          {E^(-r1) (r1^2 + bh r1), E^(-r2) (r2^2 + bh r2), E^(-r3) (r3^2 + bh r3)}};
          det = -Det[M] // FullSimplify
          subdet1 = (Cosh[r1 + r2 + r3] - Sinh[r1 + r2 + r3])
          subdet2 = det/subdet1 // FullSimplify
          subdet20 = subdet2 /. Thread[{r1, r2, r3} -> roots] /. parms;
          Plot[Im[subdet20], {mu, -100, 0}, PlotStyle -> {Thick, Black},
          PlotRange -> {-10, 10}]
          solmu1 = FindRoot[Im[subdet20] == 0, {mu, 0}]
          solmu2 = FindRoot[Im[subdet20] == 0, {mu, -7}]
          solmu3 = FindRoot[Im[subdet20] == 0, {mu, -20}]
          solmu4 = FindRoot[Im[subdet20] == 0, {mu, -40}]
          solmu5 = FindRoot[Im[subdet20] == 0, {mu, -60}]
          solmu6 = FindRoot[Im[subdet20] == 0, {mu, -80}]
          solmu7 = FindRoot[Im[subdet20] == 0, {mu, -110}]
          solmu8 = FindRoot[Im[subdet20] == 0, {mu, -150}]
          solmu9 = FindRoot[Im[subdet20] == 0, {mu, -190}]
          Subscript[mu, 1] = mu /. solmu1;
          Subscript[mu, 2] = mu /. solmu2;
          Subscript[mu, 3] = mu /. solmu3;
          Subscript[mu, 4] = mu /. solmu4;
          Subscript[mu, 5] = mu /. solmu5;
          Subscript[mu, 6] = mu /. solmu6;
          Subscript[mu, 7] = mu /. solmu7;
          Subscript[mu, 8] = mu /. solmu8;
          Subscript[mu, 9] = mu /. solmu9;
          exps = Table[roots /. parms /. {mu -> Subscript[mu, k]}, {k, 1, 9}]
          F[t, 9]





          share|cite|improve this answer











          $endgroup$



          Regarding the first DE the linear differential operator



          $$
          lambda_h delta^3 - 2 lambda_h beta_h delta^2 + left( (lambda_h beta_h - 1) beta_h - mu right) delta + beta_h^2=0
          $$



          and the three roots $delta_i(mu), i = 1,2,3$ we have that



          $$
          F(t) = sum_k C_k e^{-delta_k(mu)t}
          $$



          using now the boundary conditions



          $$
          F(0) = sum_k C_k = 0 longrightarrow (1)
          $$



          and with



          $$
          F'(0) = -sum_k C_k delta_k(mu)\
          F''(0) = sum_k C_k delta_k(mu)^2\
          $$



          giving



          $$
          -frac{sum_k C_k delta_k(mu)^2}{sum_k C_k delta_k(mu)}=beta_hlongrightarrow (2)
          $$



          and similarly



          $$
          -frac{sum_k C_k delta_k(mu)^2e^{-delta_k(mu)}}{sum_k C_k delta_k(mu)e^{-delta_k(mu)}}=beta_hlongrightarrow (3)
          $$



          then we have three linear equations $(1,2,3)$ in $C_k$ that can be arranged as



          $$
          M(mu)cdot C = 0, C = (C_k)
          $$



          This system have nontrivial solution for $det(M(mu)) = 0$ hence the roots for this determinant equation are the eigenvalues $mu_n$ and the eigenfunctions are $e^{-delta_k(mu_n)t}$



          The procedure for $G$ is quite similar.



          NOTE



          Assuming numerical values $lambda_h = 1,beta_h = -10$ we have the operator polynomial



          $$
          s^3+20s^2+(110-mu)s+100 = 0
          $$



          with roots $delta_1(mu),delta_2(mu),delta_3(mu)$



          The determinant after simplifications reads



          $$
          det(M) = left(e^{delta _1+delta _2} left(delta _1-delta _2right) delta _3 left(beta
          _h+delta _1+delta _2right) left(beta _h+delta _3right)-e^{delta _1+delta _3}
          delta _2 left(delta _1-delta _3right) left(beta _h+delta _2right) left(beta
          _h+delta _1+delta _3right)+e^{delta _2+delta _3} delta _1 left(delta _2-delta
          _3right) left(beta _h+delta _1right) left(beta _h+delta _2+delta
          _3right)right) left(cosh left(delta _1+delta _2+delta _3right)-sinh
          left(delta _1+delta _2+delta _3right)right)
          $$



          discarding $cosh (delta_1+delta_2+delta_3)-sinh
          (delta_1+delta_2+delta_3)=0$
          we follow with



          $$
          Delta(mu)=e^{delta _1+delta _2} left(delta _1-delta _2right) delta _3 left(beta _h+delta
          _1+delta _2right) left(beta _h+delta _3right)-e^{delta _1+delta _3} delta _2
          left(delta _1-delta _3right) left(beta _h+delta _2right) left(beta _h+delta
          _1+delta _3right)+e^{delta _2+delta _3} delta _1 left(delta _2-delta _3right)
          left(beta _h+delta _1right) left(beta _h+delta _2+delta _3right)=0
          $$



          and then after plotting we have



          enter image description here



          In red Re[$Delta(mu)$] and in blue Im[$Delta(mu)$]. The zeroes are the eigenvalues $mu_n$



          Attached a very basic MATHEMATICA script in order to obtain the first $mu_k$ for $lambda_h = frac 14, beta_h = -10$



          parms = {lh -> 1/4, bh -> -10};
          F[t_, n_] := Sum[
          !(*SubscriptBox[(c), ({1, j})]) Exp[exps[[j]][[1]] t] +
          !(*SubscriptBox[(c), ({2, j})]) Exp[exps[[j]][[2]] t] +
          !(*SubscriptBox[(c), ({3, j})]) Exp[exps[[j]][[3]] t], {j, 1,n}]
          sols = Solve[
          lh s^3 - 2 lh bh s^2 + ((lh bh - 1) bh - mu) s + bh^2 == 0, s] /.
          parms // FullSimplify
          roots = s /. sols;
          M = {{1, 1, 1}, {r1^2 + bh r1, r2^2 + bh r2, r3^2 + bh r3},
          {E^(-r1) (r1^2 + bh r1), E^(-r2) (r2^2 + bh r2), E^(-r3) (r3^2 + bh r3)}};
          det = -Det[M] // FullSimplify
          subdet1 = (Cosh[r1 + r2 + r3] - Sinh[r1 + r2 + r3])
          subdet2 = det/subdet1 // FullSimplify
          subdet20 = subdet2 /. Thread[{r1, r2, r3} -> roots] /. parms;
          Plot[Im[subdet20], {mu, -100, 0}, PlotStyle -> {Thick, Black},
          PlotRange -> {-10, 10}]
          solmu1 = FindRoot[Im[subdet20] == 0, {mu, 0}]
          solmu2 = FindRoot[Im[subdet20] == 0, {mu, -7}]
          solmu3 = FindRoot[Im[subdet20] == 0, {mu, -20}]
          solmu4 = FindRoot[Im[subdet20] == 0, {mu, -40}]
          solmu5 = FindRoot[Im[subdet20] == 0, {mu, -60}]
          solmu6 = FindRoot[Im[subdet20] == 0, {mu, -80}]
          solmu7 = FindRoot[Im[subdet20] == 0, {mu, -110}]
          solmu8 = FindRoot[Im[subdet20] == 0, {mu, -150}]
          solmu9 = FindRoot[Im[subdet20] == 0, {mu, -190}]
          Subscript[mu, 1] = mu /. solmu1;
          Subscript[mu, 2] = mu /. solmu2;
          Subscript[mu, 3] = mu /. solmu3;
          Subscript[mu, 4] = mu /. solmu4;
          Subscript[mu, 5] = mu /. solmu5;
          Subscript[mu, 6] = mu /. solmu6;
          Subscript[mu, 7] = mu /. solmu7;
          Subscript[mu, 8] = mu /. solmu8;
          Subscript[mu, 9] = mu /. solmu9;
          exps = Table[roots /. parms /. {mu -> Subscript[mu, k]}, {k, 1, 9}]
          F[t, 9]






          share|cite|improve this answer














          share|cite|improve this answer



          share|cite|improve this answer








          edited Jan 19 at 18:38

























          answered Jan 11 at 21:58









          CesareoCesareo

          8,6793516




          8,6793516












          • $begingroup$
            Thanks for the explanation. I guess you used $F'/F''$ while writing $(2)$ and $(3)$ while the BC are actually $F''/F'$
            $endgroup$
            – Indrasis Mitra
            Jan 12 at 3:10










          • $begingroup$
            Also will solving for $det(M(mu))=0$ take care of all the possible values of $mu$ that needs to be considered i.e. $mu>0$,$mu=0$ and $mu<0$ ? for finding the eigenvalues ?
            $endgroup$
            – Indrasis Mitra
            Jan 12 at 4:26










          • $begingroup$
            my last comment is pretty ignorant. So actually $F(t)=C_1e^{-delta_1(mu)t}+C_2e^{-delta_2(mu)t}+C_3e^{-delta_3(mu)t}$, for the three roots of the characteristic equation. Am i right ?
            $endgroup$
            – Indrasis Mitra
            Jan 12 at 6:40












          • $begingroup$
            I followed the steps you suggested to atrrive at $mathbb{det}(M(mu))=0$. I find that it has $delta_1(mu)$,$delta_2(mu)$ and $delta_3(mu)$. I have edited the original question to reflect my attempt. I cannot figure out how to proceed further, am i doing something wrong ?
            $endgroup$
            – Indrasis Mitra
            Jan 13 at 3:45










          • $begingroup$
            @IndrasisMitra See attached note.
            $endgroup$
            – Cesareo
            Jan 13 at 9:19


















          • $begingroup$
            Thanks for the explanation. I guess you used $F'/F''$ while writing $(2)$ and $(3)$ while the BC are actually $F''/F'$
            $endgroup$
            – Indrasis Mitra
            Jan 12 at 3:10










          • $begingroup$
            Also will solving for $det(M(mu))=0$ take care of all the possible values of $mu$ that needs to be considered i.e. $mu>0$,$mu=0$ and $mu<0$ ? for finding the eigenvalues ?
            $endgroup$
            – Indrasis Mitra
            Jan 12 at 4:26










          • $begingroup$
            my last comment is pretty ignorant. So actually $F(t)=C_1e^{-delta_1(mu)t}+C_2e^{-delta_2(mu)t}+C_3e^{-delta_3(mu)t}$, for the three roots of the characteristic equation. Am i right ?
            $endgroup$
            – Indrasis Mitra
            Jan 12 at 6:40












          • $begingroup$
            I followed the steps you suggested to atrrive at $mathbb{det}(M(mu))=0$. I find that it has $delta_1(mu)$,$delta_2(mu)$ and $delta_3(mu)$. I have edited the original question to reflect my attempt. I cannot figure out how to proceed further, am i doing something wrong ?
            $endgroup$
            – Indrasis Mitra
            Jan 13 at 3:45










          • $begingroup$
            @IndrasisMitra See attached note.
            $endgroup$
            – Cesareo
            Jan 13 at 9:19
















          $begingroup$
          Thanks for the explanation. I guess you used $F'/F''$ while writing $(2)$ and $(3)$ while the BC are actually $F''/F'$
          $endgroup$
          – Indrasis Mitra
          Jan 12 at 3:10




          $begingroup$
          Thanks for the explanation. I guess you used $F'/F''$ while writing $(2)$ and $(3)$ while the BC are actually $F''/F'$
          $endgroup$
          – Indrasis Mitra
          Jan 12 at 3:10












          $begingroup$
          Also will solving for $det(M(mu))=0$ take care of all the possible values of $mu$ that needs to be considered i.e. $mu>0$,$mu=0$ and $mu<0$ ? for finding the eigenvalues ?
          $endgroup$
          – Indrasis Mitra
          Jan 12 at 4:26




          $begingroup$
          Also will solving for $det(M(mu))=0$ take care of all the possible values of $mu$ that needs to be considered i.e. $mu>0$,$mu=0$ and $mu<0$ ? for finding the eigenvalues ?
          $endgroup$
          – Indrasis Mitra
          Jan 12 at 4:26












          $begingroup$
          my last comment is pretty ignorant. So actually $F(t)=C_1e^{-delta_1(mu)t}+C_2e^{-delta_2(mu)t}+C_3e^{-delta_3(mu)t}$, for the three roots of the characteristic equation. Am i right ?
          $endgroup$
          – Indrasis Mitra
          Jan 12 at 6:40






          $begingroup$
          my last comment is pretty ignorant. So actually $F(t)=C_1e^{-delta_1(mu)t}+C_2e^{-delta_2(mu)t}+C_3e^{-delta_3(mu)t}$, for the three roots of the characteristic equation. Am i right ?
          $endgroup$
          – Indrasis Mitra
          Jan 12 at 6:40














          $begingroup$
          I followed the steps you suggested to atrrive at $mathbb{det}(M(mu))=0$. I find that it has $delta_1(mu)$,$delta_2(mu)$ and $delta_3(mu)$. I have edited the original question to reflect my attempt. I cannot figure out how to proceed further, am i doing something wrong ?
          $endgroup$
          – Indrasis Mitra
          Jan 13 at 3:45




          $begingroup$
          I followed the steps you suggested to atrrive at $mathbb{det}(M(mu))=0$. I find that it has $delta_1(mu)$,$delta_2(mu)$ and $delta_3(mu)$. I have edited the original question to reflect my attempt. I cannot figure out how to proceed further, am i doing something wrong ?
          $endgroup$
          – Indrasis Mitra
          Jan 13 at 3:45












          $begingroup$
          @IndrasisMitra See attached note.
          $endgroup$
          – Cesareo
          Jan 13 at 9:19




          $begingroup$
          @IndrasisMitra See attached note.
          $endgroup$
          – Cesareo
          Jan 13 at 9:19


















          draft saved

          draft discarded




















































          Thanks for contributing an answer to Mathematics Stack Exchange!


          • Please be sure to answer the question. Provide details and share your research!

          But avoid



          • Asking for help, clarification, or responding to other answers.

          • Making statements based on opinion; back them up with references or personal experience.


          Use MathJax to format equations. MathJax reference.


          To learn more, see our tips on writing great answers.




          draft saved


          draft discarded














          StackExchange.ready(
          function () {
          StackExchange.openid.initPostLogin('.new-post-login', 'https%3a%2f%2fmath.stackexchange.com%2fquestions%2f3069768%2feigen-values-of-a-third-order-linear-homogenous-ode%23new-answer', 'question_page');
          }
          );

          Post as a guest















          Required, but never shown





















































          Required, but never shown














          Required, but never shown












          Required, but never shown







          Required, but never shown

































          Required, but never shown














          Required, but never shown












          Required, but never shown







          Required, but never shown







          Popular posts from this blog

          Can a sorcerer learn a 5th-level spell early by creating spell slots using the Font of Magic feature?

          Does disintegrating a polymorphed enemy still kill it after the 2018 errata?

          A Topological Invariant for $pi_3(U(n))$